Use the estimated angle to find the distance of the life-raft from the lighthouse.

Answers

Answer 1

The correct angle of elevation for the lighthouse's top is roughly 1.91°.

How to find the distance of the lift-raft from the lighthouse?

Consider the diagram drawn below, P is the position of the person in the lift-raft, B is the base of the lighthouse and the T is top of the lighthouse.

The diagram shows that the angle produced between the person in the life raft's line of sight to the top of the lighthouse and the horizontal is equal to the angle formed by the top of the lighthouse's height. Call this angle "[tex]\alpha[/tex]".

a) The distance "d" between the life raft and the lighthouse can be calculated using trigonometry.

[tex]tan\alpha = \frac{opposite}{adjacent} \\tan\alpha = \frac{20}{d}[/tex]

putting the value of [tex]\alpha[/tex] = 3 degrees put into the above equation.

[tex]tan3 = \frac{20}{d}[/tex]

[tex]d = \frac{20}{tan3}[/tex]

d = 354.14 (rounded to 2 decimal points

The life raft is therefore roughly 354.14 meters away from the lighthouse.

b)

Let's now calculate the proper angle of elevation for the situation when the life raft is 600 meters from the lighthouse.

[tex]tan\alpha = \frac{opposite}{adjacent}\\ tan\alpha = \frac{20}{600}[/tex]

solving for [tex]\alpha[/tex] we get,

[tex]\alpha = tan^{-1} (\frac{20}{100})\\\alpha = 1.91 degree\\[/tex]

Since the life raft is 600 meters from the lighthouse, the correct angle of elevation for the lighthouse's top is roughly 1.91°.

Learn more about the angle of elevation here:

https://brainly.com/question/16716174

#SPJ1

The complete question in the form of the image below:

Use The Estimated Angle To Find The Distance Of The Life-raft From The Lighthouse.
Use The Estimated Angle To Find The Distance Of The Life-raft From The Lighthouse.

Related Questions

please answer i need help!!!!!!!!!!!!!!

Answers

Answer: 29

Step-by-step explanation:

<F is 29 degrees because 180-58=122 and 180-122= 58, so then you divide 58 by 2 to get 29 for each x. The x's are each equal to 29.

Answer:

29 degrees

Step-by-step explanation:

The problem shows angle HGE as 58 degrees.

Segment HF is 180 degrees so we can subtract 58 from 180 to get 122 degrees for angle g

Now, the sum of all angles in a triangle is 180.

So, there are 3 angles in the triangle:

g(we know this is 122)

e

f

the graph denotes that the measures of angle e and f are the same because they are the same variable.

Lets subtract 122 from 180 to get 58.

So, angle e and f together combine to make 58 degrees.

But we also know that e and f have the same measure.

So, to find the measure of f, simply divide 58/2 to get 29 degrees.

May I please have Brainliest? I put a lot of thought and effort into my answers, so it would be much appreciated!

CNNBC recently reported that the mean annual cost of auto insurance is 957 dollars. Assume the standard deviation is 193 dollars. You will use a simple random sample of 58 auto insurance policies. You may assume original population is approximatley normally distributed, and round your answers to three decimals.

Find the probability that a single randomly selected policy has a mean value between 964.6 and 1000.1 dollars.
P(964.6 < X < 1000.1) = ???

Find the probability that a random sample of size
has a mean value between 964.6 and 1000.1 dollars.
P(964.6 < M < 1000.1) = ???

Answers

The probabilities are given as follows:

P(964.6 < X < 1000.1) = 0.071 = 7.1%.P(964.6 < M < 1000.1) = 0.338 = 33.8%

How to obtain probabilities using the normal distribution?

The z-score of a measure X of a normally distributed variable that has mean represented by [tex]\mu[/tex] and standard deviation represented by [tex]\sigma[/tex] is obtained by the equation presented as follows:

[tex]Z = \frac{X - \mu}{\sigma}[/tex]

The z-score represents how many standard deviations the measure X is above or below the mean of the distribution of the data-set, depending if the obtained z-score is positive(above the mean) or negative(below the mean).The z-score table is used to obtain the p-value of the z-score, and it represents the percentile of the measure X in the distribution.By the Central Limit Theorem, the sampling distribution of sample means of size n has standard deviation given by the equation presented as follows: [tex]s = \frac{\sigma}{\sqrt{n}}[/tex].

The mean and the standard deviation for this problem is given as follows:

[tex]\mu = 957, \sigma = 193[/tex]

The probability is the p-value of Z when X = 1000.1 subtracted by the p-value of Z when X = 964.6, hence:

Z = (1000.1 - 957)/193

Z = 0.22.

Z = 0.22 has a p-value of 0.5871.

Z = (964.6 - 957)/193

Z = 0.04.

Z = 0.04 has a p-value of 0.5160.

Hence:

0.5871 - 0.5160 = 0.0711.

For the sample of 58, the standard error is obtained as follows:

s = 193/sqrt(58) = 25.34.

Hence:

Z = (1000.1 - 957)/25.34

Z = 1.7.

Z = 1.7 has a p-value of 0.9554.

Z = (964.6 - 957)/25.34

Z = 0.3.

Z = 0.3 has a p-value of 0.6179.

Hence:

0.9554 - 0.6179 = 0.338.

More can be learned about the normal distribution at https://brainly.com/question/25800303

#SPJ1

Transactions on Furnell's credit card are shown for the month of June. The interest rate is 1.4% per month.

June 1 Balance $352.12
June 4 Sears $331.89
June 8 eBay $81.58
June 15 Outback $30
June 18 Payment $200




Find the average daily balance $
184.89

Find the interest for the month $

Find the balance for the following month $

Answers

The interest for the month is:  $2.59

The balance for the month is: $598.18.

We have the information from the question is:

The interest rate is 1.4% per month.

June 1 Balance $352.12

June 4 Sears $331.89

June 8 eBay $81.58

June 15 Outback $30

June 18 Payment $200

Now, According to the question:

The average daily balance for June is $184.89.

To calculate the interest for the month,

Multiply the average daily balance by the interest rate:

$184.89 × 1.4% = $2.59.

To find the balance for the month :

Add the initial balance, transactions, and interest, then subtract the payment:

$352.12 + $331.89 + $81.58 + $30 + $2.59 - $200 = $598.18.

Learn more about Interest Rate at:

https://brainly.com/question/13324776

#SPJ1

Assume the random variable x is distributed with mean of 50 and a standard deviation of 7. Compute the probability.
P(x > 38)

Answers

Using the given random variable and the mean the required probability in the given situation is 0.9772.

What is probability?

A probability is a numerical representation of the likelihood or chance that a specific event will take place.

Both proportions ranging from 0 to 1 and percentages ranging from 0% to 100% can be used to describe probabilities.

= P(x>36) = P(X-μ/σ>36-50/7)

= P(Z>-14/7) Z=X-μ/σ

= P(Z>-2)

= P(Z<2)  [P(Z<z) = P(Z>-z)]

= 0.9772 by the p-value table

Therefore, using the given random variable and the mean the required probability in the given situation is 0.9772.

Know more about probability here:

https://brainly.com/question/24756209

SPJ1

Correct question:

Assume the random variable x is normally distributed with mean 50 and a standard deviation 7. Find the indicated probability. P(X>36)

the set of five number each of which is divisble by 3

Answers

Answer:

Here's a set of five numbers, each of which is divisible by 3:

{ 3, 6, 9, 12, 15 }

All the numbers in this set are multiples of 3, so they are all divisible by 3.

Enter the endpoints as ordered pairs (x, y).
Provide your answer below:
The endpoints of the latus rectum are and
(y + 3)² = (x - 2)

Answers

The endpoints of the latus rectum of the parabola are (3, -2) and (3, -4)

Given data ,

Let the equation of the parabola be (y + 3)² = (x - 2)

Now , y² + 6y + 9 = x - 2

y² + 6y = x - 11

Now we can see that the vertex of the parabola is (2, -3), and the axis of symmetry is parallel to the y-axis. This means that the focus and directrix will also be parallel to the y-axis.

x = h - p

where (h, k) is the vertex and "p" is the distance between the vertex and the focus. In this case, (h, k) = (2, -3) and p = 1, so

x = 2 - 1

x = 1

Therefore, the equation of the directrix is x = 1

And , the axis of symmetry of the parabola is parallel to the y-axis, the latus rectum will be parallel to the directrix, and its endpoints will be equidistant from the focus and the directrix

The distance between the focus and the directrix is 1 unit, so the distance between the endpoints of the latus rectum will also be 1 unit

Therefore, the endpoints of the latus rectum will be at the points where the perpendiculars drawn from the focus to the directrix intersect the parabola

To find the points of intersection between this line and the parabola, we can substitute x = 3 into the equation of the parabola and solve for y

(y + 3)² = (x - 2)

(y + 3)² = (3 - 2)

(y + 3)² = 1

y + 3 = ±1

y = -2 or y = -4

Hence , the points of intersection between the perpendicular and the parabola are (3, -2) and (3, -4)

To learn more about parabola click :

https://brainly.com/question/24042022

#SPJ1

Please answer if you know this one with the steps thank you.

Answers

Answer:  36600 = E

Step-by-step explanation:

Given:

T=.2 (E - 10600)      

T=5200  

Find:  E

Solution:  

>substitute into thequation.  You know T=5200

5200 = .2(E-10600)     > Divide both sides by .2

26000 = E- 10600       > add 10600 to both sides

36600 = E                    

using calculations show that the height of the barrel of oil is 96.82cm

Answers

The use of calculations to show that the height of the barrel of oil is 96.82cm is given below:

The volume is given as V = 42 gal

The radius is given as r = 18/2 = 9 in

The height is given as h= 96.83 cm

How to solve

Using the formula:

V = πr²h

h = V/(πr²)

The volume is given as V = 42 gal

42 gal x 3.7854 l/gal = 158.987 l

158.987 l = 158,987 ml = 158,987 cm³

The radius is given as r = 18/2 = 9 in

9 in x 2.54 cm/in = 22.86 cm

The height is given as h = (158987 cm³) / π(22.86 cm)² ≈ 96.82 cm

depending on how you round, the more precise answer is 96.8285 ≈ 96.83 cm

Read more about height here:

https://brainly.com/question/1739912

#SPJ1

Can someone help me please

Answers

The matrix formed by performing the row operation 2R₁ + R₂ — R₂ on M will have a new R₂ = [ -8 -1 -3]

What is the row of a matrix

A rectangular array of numbers or mathematical objects which are arranged in rows and columns is called a matrix. Each row of a matrix is a horizontal sequence of numbers or objects that are separated by commas and enclosed within square brackets, and it represents a vector in the row space of the matrix.

performing the row operation 2R₁ + R₂ — R₂ on M, we have;

2(-3) + (-2) = -8 {row 2 column 1}

2(-1) + 1 = -1 {row 2 column 2}

2(-4) + 5 = -3 {row 2 column 3}

Therefore, the matrix formed by performing the row operation 2R₁ + R₂ — R₂ on M will have a new R₂ = [ -8 -1 -3]

Read more about matrix row here:https://brainly.com/question/1279486

#SPJ1

Fully factorise 3h² + 12h​

Answers

Answer: To factorize 3h² + 12h, we can first find the greatest common factor (GCF) of the two terms, which is 3h:

3h(h + 4)

This is the fully factorized form of 3h² + 12h.

What is the equation of a parabola with a vertical axis, vertex (h, k), and directrix y = k – p, where p is a nonzero real number? How can the equation be simplified if the vertex is at the origin?

Answers

The equation of a parabola with a vertical axis and vertex (h, k) is given by:

(x - h)² = 4p(y - k)

How to explain the equation

In the equation, where p is the distance from the vertex to the focus (and also the distance from the vertex to the directrix).

If the vertex is at the origin (h=0, k=0), then the equation simplifies to:

x² = 4py

where p is still the distance from the vertex to the focus (and also the distance from the vertex to the directrix).

Learn more about equations on

https://brainly.com/question/2972832

#SPJ1

On Monday Harriet ate 1/4 of a 8 slice of pizza on Tuesday she ate 1/2 of the same pizza what fraction of The Whole Pizza did she eat

Answers

Harriet ate a total of 3/4 of the total pizza

Given data ,

On Monday Harriet ate 1/4 of a 8 slice of pizza

And , on Tuesday she ate 1/2 of the same pizza

Now , If the pizza has 8 slices, then Harriet ate 1/4 of 8 slices on Monday, which is 2 slices.

On Tuesday, Harriet ate 1/2 of the same pizza, which is 4 slices.

So, in total, Harriet ate 2 + 4 = 6 slices of pizza

On simplifying the equation , we get

The whole pizza has 8 slices, so Harriet ate 6/8 of the pizza, which simplifies to 3/4.

Hence , Harriet ate 3/4 of the whole pizza

To learn more about equations click :

https://brainly.com/question/19297665

#SPJ1

There are 8 blue marbles, 6 red marbles, 2 green marbles and 4 black marbles in a box.
What is the probability that the marble is not green?

Answers

the probability that the marble is not green is 0.9 or 90%. This means that if we were to randomly select a marble from the box, there is a 90% chance that it will not be green.

How to solve probability?

To calculate the probability that a marble is not green, we need to first determine the total number of marbles in the box, which is given by:

Total number of marbles = 8 + 6 + 2 + 4 = 20

Next, we need to determine the number of marbles that are not green. Since there are 2 green marbles, the number of marbles that are not green is given by:

Number of marbles that are not green = 20 - 2 = 18

Finally, we can calculate the probability of selecting a marble that is not green by dividing the number of marbles that are not green by the total number of marbles in the box, as follows:

Probability of selecting a marble that is not green = Number of marbles that are not green / Total number of marbles

= 18 / 20

= 0.9 or 90%

Therefore, the probability that the marble is not green is 0.9 or 90%. This means that if we were to randomly select a marble from the box, there is a 90% chance that it will not be green.

To know more about probability visit :-

https://brainly.com/question/13604758

#SPJ1

Compare each pair of expressions using >, <, or =.
.-32
. |-32|
5 -5
15
___|15|
. |5|_____|-5|
2-17

2 ____ |-17|
. |-27|_____|-45|
.-27______-45

Answers

Comparing each pair of expressions using >, <, or = is given below:

15 > |___15| (because |___15| is equal to 15)2 - 17 < ▾ (because 2 - 17 equals -15, which is less than the square root symbol)-27 > -45 (because -27 is closer to zero than -45)

'How to solve

-32 < |-32| (because -32 is negative and |-32| is positive)

5 - 5 = 0 (because subtracting the same number results in zero)

15 > |___15| (because |___15| is equal to 15)

|5| = |___|-5|| (because both expressions are equal to 5)

2 - 17 < ▾ (because 2 - 17 equals -15, which is less than the square root symbol)

2 > |____|-17|| (because 2 is positive and |-17| is also positive)

|-27| > ||-45|| (because |-27| is 27 and ||-45|| is 45)

-27 > -45 (because -27 is closer to zero than -45)

Read more about expressions here:

https://brainly.com/question/1859113
#SPJ1

please help me for 50 points!!

simplify: -3 √84x^3

A. -6x√21x
B. -6√21
C. 6x√21x

Answers

Answer:

C

Step-by-step explanation:

the prime factorization of 84 is 2 x2 x 3 x 7

I can rewrite the problem

-3[tex]\sqrt{84x^{3} }[/tex]

-3[tex]\sqrt{(2)(2)(3)(7)xxx}[/tex]  pull out the pairs

-3(2)x[tex]\sqrt{(3)(7)x}[/tex]

-6x[tex]\sqrt{21x}[/tex]

Helping in the name of Jesus.

The hypotenuse of an isosceles right triangle is 13 centimeters longer than either of its legs. Find the exact length of
each side. (Hint: An isosceles right triangle is a right triangle whose legs are the same length.)
The length of one leg is
the length of the other leg is
(Simplify your answers. Type exact answers, using radicals as needed.)
and the length of the hypotenuse is

Answers

The length of the other legs of the isosceles right triangle is 13 + 13√2 centimetres.

How to find the sides of a right triangle?

The hypotenuse of an isosceles right triangle is 13 centimetres longer than either of its legs.

An isosceles right triangle is a right triangle whose legs are the same length. The sides of a right triangle can be found using Pythagoras's theorem.

Therefore,

let

y = other legs

Therefore,

(x + 13)² = x² + x²

(x + 13)(x + 13) = 2x²

x² + 13x + 13x + 169 = 2x²

x² - 26x - 169 = 0

Therefore, using the quadratic formula,

[tex]\frac{-b+\sqrt{b^{2} - 4ac } }{2a}[/tex]

where

a = 1b = -26c = -169

x = 13 ± 13√2

Therefore, we can only use positive values

x = 13 + 13√2

learn more on right triangle here: https://brainly.com/question/22406443

#SPJ1

A pair of standard dice are rolled. Find the
probability of rolling a sum of 7 with these dice.

Answers

Answer:

6 out of 36 or 1 out of 6

Step-by-step explanation:

There are 36 possible ways two dice can roll, so the probability of the sum of seven is 6 out of 36, or 1/6.

Answer:

[tex]\boxed{\sf 16.67\%}.[/tex]

Step-by-step explanation:

1. Determine the total amount of different outcomes.

So since we have a pair of dice, assuming they are standard 6 faced dices, the total amount of outcomes is given by just multiplying the number of faces or different numbers that the dices can present:

6 x 6 = 36 possible outcomes.

2. Determine the amout of outcomes that meet the requirement of adding up to 7.

Now, here we need to count all the possible cases where the outcome of both dices will equal 7. And there are said cases:

1 + 6 = 7

Explanation. Here the first dice presented number 1 and the second presented number 6, adding up to 7.

Other cases that meet the requirement:

2 + 5 = 7

3 + 4 = 7

4 + 3 = 7

5 + 2 = 7

6 + 1 = 7

So the total amount of events that will meet the requirement is 6.

3. Find the probability.

So if we have an event with 36 possible outcomes, and only 6 of those 36 outcomes will mean a success on the experiment. Then, to find the probability of those 6 events from happening all we need to do is divide the amount of events considered "success" which is the numbers adding up to 7 by the total amout of outcomes (36).

So, the probability of rolling a sum of 7 with 2 dices is:

[tex]\sf P(\dfrac{6}{36} )=\dfrac{1}{6} =\boxed{\sf 16.67\%}.[/tex]

-------------------------------------------------------------------------------------------------------

Learn more about probability calculations here:

brainly.com/question/29779138

ASAP
A rectangular prism is shown in the image.

A rectangular prism with dimensions of 3 yards by 5 yards by 5 and one half yard.

What is the volume of the prism?

forty one and one fourth yd3
fifty six and one fourth yd3
eighty two and one half yd3
165 yd3

Answers

As a result, the prism's volume is given by V = lwh = 3 x 5 x 5.5 = 82.5 cubic yards. Thus, option C—eighty two and a half yards—is the correct response.

what is volume ?

The quantity of room an object takes up in three dimensions is measured by its volume. It is frequently expressed in cubic units like cubic metres, cubic feet, and cubic centimetres. Mathematics formulas that are particular to the shape of the object can be used to calculate its volume, such as V = l w h for a rectangular shape or V = (4/3)r3 for a sphere.

given

V = lwh is the formula for calculating the volume of a rectangular prism, where l, w, and h stand for the prism's length, width, and height, respectively.

Here, the dimensions are 3 yards in length, 5 yards in breadth, and 5.5 yards in height.

As a result, the prism's volume is given by V = lwh = 3 x 5 x 5.5 = 82.5 cubic yards.

Thus, option C—eighty two and a half yards—is the correct response.

To know more about volume visit :-

https://brainly.com/question/1578538

#SPJ1

Simplify using law of exponents. (3^4)^8

Answers

Step-by-step explanation:

[tex] {( ({3})^{4} })^{8} \\ = {3}^{32} [/tex]

TRIG QUESTIONS PLS HELP

Answers

5. The answer is option D (12.53, 61.39°)

6. Option D. ((7√2)/2, (-7√2)/2)

7. Option C. (104.40, -16.70°)

How did we get the values?

Question 5:

Apply the Pythagorean theorem to find the distance r from the fire station to the destination:

r = √(62 + 11²) = 12.53 (rounded to the nearest hundredth)

The angle θ between the positive x-axis and the line connecting the fire station to the destination is found using tangent:

tan(θ) = ¹¹/₆

θ = arctan(¹¹/₆) = 61.39° (rounded to the nearest hundredth)

Therefore, the answer is (12.53, 61.39°), which is option D.

Answer: D. (12.53, 61.39°)

Question 6:

Convert from polar coordinates to rectangular coordinates using the following formulas:

x = r cos(θ)

y = r sin(θ)

Plugging in the values, we get:

x = -7 cos(3π/4) = -7√(2)/2) = 3.5√(2)

y = -7 sin(3π/4) = -7√(2)/2) = -3.5√(2)

Therefore, the rectangular coordinates of the point are (3.5sqrt(2), -3.5sqrt(2)), which can also be written as (-4.95, -4.95) (rounded to the nearest hundredth).

Answer: (7√(2)/2), -7√(2)/2))

Question 7:

Use the Pythagorean theorem to find the distance r from the starting point to the current position:

r = √(100² + 30²) = 104.40 (rounded to the nearest hundredth)

The angle θ between the positive x-axis and the line connecting the starting point to the current position can be found using tangent:

tan(θ) = 30/100

θ = arctan(30/100) = 16.70° (rounded to the nearest hundredth)

However, since the woman is south of the starting point, the angle should be in the negative direction, so we have:

θ = -16.70°

Therefore, the answer is (104.40, -16.70°), which is option C.

Answer: C. (104.40, -16.70°)

learn more about Pythagorean theorem: https://brainly.com/question/231802

#SPJ1

PLEASE HELPPPPP MEHHHH

A composite figure is composed of a semicircle whose radius measures 5 inches added to a square whose side measures 10 inches. A point within the figure is randomly chosen.

What is the probability that the randomly selected point is in the semicircular region?

Enter your answer rounded to the nearest tenth in the box.

%

Answers

To solve this problem, we need to find the total area of the composite figure and the area of the semicircular region, and then divide the area of the semicircular region by the total area to get the probability that the randomly selected point is in the semicircular region.

The area of a semicircle with radius r is (1/2)πr^2, and the area of a square with side s is s^2. Therefore, the total area of the composite figure is:

Total area = (1/2)π(5^2) + 10^2 = 25π/2 + 100

The area of the semicircular region is (1/2)π(5^2) = 25π/2.

The probability of selecting a point in the semicircular region is:

Probability = Area of semicircular region / Total area
Probability = (25π/2) / (25π/2 + 100)
Probability = 0.198 (rounded to the nearest tenth)

Therefore, the probability that the randomly selected point is in the semicircular region is approximately 0.2, or 20%.

1. A survey conducted by a national research center asked a random sample of 920
teenagers in the United States how often they use a video streaming service.
From the sample, 59% answered that they use a video streaming service every day.
a. Construct and interpret a 95% confidence interval for the proportion of all
teenagers in the United States who would respond that they use a video
streaming service every day.
b. Based on the confidence interval in part (a), do the sample data provide
convincing statistical evidence that the proportion of all teenagers in the United
States who would respond that they use a video streaming service every day is
not 0.5? Justify your answer.


Mean Standard Deviation Sample Size
Standard care 0.57 0.26 56
New treatment 0.69 0.27 56

2. Patients experiencing symptoms of a heart attack are routinely transported to a
hospital in an ambulance. In a study of a new treatment thought to reduce damage to
the heart, patients experiencing symptoms of a heart attack were randomly assigned to
one of two groups. During transportation to the hospital, patients in one group received
standard care, and patients in the other group received the new treatment consisting of
standard care and the application of a blood pressure cuff.
The response variable measured for each patient was a number between 0 and 1,
referred to as the myocardial salvage index (MSI). A higher MSI value indicates a more
positive outcome for the patient. Summary statistics for the MSI responses of the two
groups are shown in the table.
Do the data provide convincing statistical evidence that the new treatment results in a
higher mean MSI value than does the standard care among people similar to the
patients in the study?

Answers

Since the calculated t-value is greater than the critical t-value, we reject the null hypothesis, concluding that there's convincing evidence that the new treatment results in a higher mean MSI value than standard care.

How to solve

a. The 95% confidence interval for the proportion of teenagers using video streaming services daily is (0.5579, 0.6221). This means we're 95% confident that 55.79% to 62.21% of US teenagers use such services daily.

b. The results demonstrate that the proportion is not 0.5 because the full confidence interval is higher than 0.5.2. We obtain a t-value of roughly 2.51 with 108.52 degrees of freedom using a two-sample t-test.

At a significance level of 0.05, the critical t-value for a one-tailed test is approximately 1.66.

Since the calculated t-value is greater than the critical t-value, we reject the null hypothesis, concluding that there's convincing evidence that the new treatment results in a higher mean MSI value than standard care.

Read more about confidence interval here:

https://brainly.com/question/15712887

#SPJ1

Find to value of a if you knew the perimeter was 250 inches long

Answers

Answer:

To find the value of "a" when you know the perimeter of a shape, you need to know the specific shape you're referring to. Please provide more information about the shape in question, such as whether it's a rectangle, triangle, or another geometric figure. Additionally, if you have any specific measurements or relationships between the sides of the shape, please provide them as well.

The graph shows the area in square feet Sally has left to paint. The equation A=392-14x gives the square feet Lisa has left to paint as a function of time in minutes. Which statements are true?

Answers

The statements that are true include the following:

B. Lisa paints more square feet per minute than Sally.

C. Lisa will finish painting before Sally.

D. After 10 minutes of painting Sally will have less to paint than Lisa.

How to determine an equation of this line?

In Mathematics and Geometry, the point-slope form of a straight line can be calculated by using the following mathematical expression:

y - y₁ = m(x - x₁)

Where:

x and y represent the data points.m represent the slope.

First of all, we would determine the slope of this line;

Slope (m) = (y₂ - y₁)/(x₂ - x₁)

Slope (m) = (0 - 360)/(30 - 0)

Slope (m) = -360/30

Slope (m) = -12

At data point (0, 360) and a slope of -12, a linear equation for this line can be calculated by using the point-slope form as follows:

y - y₁ = m(x - x₁)

y - 360 = -12(x - 0)  

y = -12x + 360

A = 360 - 12x

For Sally, the time to finish;

360 = 12x

x = 360/12 = 30 min.

For Lisa, the time to finish;

392 = 14x

x = 392/14 = 28 min (Lisa will finish painting before Sally)

For Sally, when x = 10;

A = 360 - 12(10)

A = 240 sq. feet.

For Lisa, when x = 10;

A = 392 - 14(10)

A = 252 sq. feet.

For Sally, when x = 20;

A = 360 - 12(20)

A = 120 sq. feet.

For Lisa, when x = 20;

A = 392 - 14(20)

A = 112 sq. feet.

Read more on point-slope here: brainly.com/question/24907633

#SPJ1

A gas station sells regular gas for $2.20 per gallon and premium gas for $2.75 a gallon. At the end of a business day 340 gallons of gas had been sold, and receipts totaled $803. How many gallons of each type of gas had been sold?

Answers

Using a system of equations, the quantities of each type of gas sold by the gas station for the business day are as follows:

Regular Gas = 240 gallonsPremium Gas = 100 gallons.

What is a system of equations?

A system of equations refers to two or more equations solved concurrently.

A system of equations is also known as simultaneous equations because they can be solved at the same time.

                              Regular    Premium

Price per gallon      $2.20       $2.75

Total number of gallons of gas sold = 340

The total receipts for the 340 gallons = $803.

Let the number of gallons of the regular gas = x

Let the number of gallons of the premium gas = y

Equations:

x + y = 340 ...Equation 1

2.2x + 2.75y = 803 ...Equation 2

Multiply Equation 1 by 2.2:

2.2x + 2.2y = 748 ...Equation 3

Subtract Equation 3 from Equation 2:

2.2x + 2.75y = 803

-

2.2x + 2.2y = 748

0.55y = 55

y = 100

x = 240 (340 - 100)

Learn more about a system of equations at https://brainly.com/question/13729904.

#SPJ1

P(RI-T) = 0.25, P(-R|T) = 0.2, P(T) = 0.1
What is P(R)?

O 0.245
O insufficient information
O 0.1060
O 0.305

Answers

Answer:

We can use Bayes' theorem to calculate P(R), which states that the probability of an event A given event B is equal to the probability of B given A multiplied by the probability of A, divided by the probability of B:

P(R|T) = P(T|R) * P(R) / P(T)

We can rearrange this equation to solve for P(R):

P(R) = P(R|T) * P(T) / P(T|R)

We are given that P(RI-T) = 0.25, which can be written as:

P(R∩T) = P(R|T) * P(T) = 0.25

We are also given that P(-R|T) = 0.2, which can be written as:

P(-R∩T) = P(-R|T) * P(T) = 0.2 * 0.1 = 0.02

We can use the law of total probability to find P(T|R):

P(T|R) = P(RI-T) / P(R) = 0.25 / P(R)

We can substitute these values into the equation for P(R) to get:

P(R) = P(R|T) * P(T) / P(T|R)

= 0.25 / [P(T|R) * P(T) + P(-R|T) * P(T)]

= 0.25 / [0.25 + 0.02]

= 0.1060

Therefore, the answer is option C: 0.1060.

find surface area of rectangular prism 2.5 2 14 1.5

Answers

The surface area of the rectangular prism is 24.62 square units

What is the surface area of the rectangular prism?

From the question, we have the following parameters that can be used in our computation:

2.5 by 2.14 by 1.5

The surface area of the rectangular prism is calculated as

Area = 2 * (Length * Width + Length * Height + Height  * Width )

substitute the known values in the above equation, so, we have the following representation

Area = 2 * (2.5 * 2.14 + 2.5 * 1.5 + 2.14 * 1.5)

Evaluate

Area = 24.62

Hence, the surface area is 24.62 square units

Read more about surface area at

brainly.com/question/26403859

#SPJ1

Put the expressions in order from the least to greatest

Answers

Step-by-step explanation:

[tex] { ({4}^{ - 4} )}^{3} \\ = {4}^{ - 12} [/tex]

[tex] \frac{ {4}^{15} }{ {4}^{5} } \\ = {4}^{10} [/tex]

[tex] {4}^{ - 5} \times {4}^{ - 4} \\ = {4}^{ - 9} [/tex]

[tex] \frac{1}{ {4}^{ - 12} } \\ = {4}^{12} [/tex]

least to greatest

[tex]1)\: {4}^{ - 12} \\ 2) \: {4}^{ - 9} \\ 3) \: {4}^{10 } \\ 4) \: {4}^{12} [/tex]

#CMIIW

find the statement that is incorrect. Then, correct and rewrite the statement in the space provided. Show any necessary work.

Answers

The incorrect statement is

The transformation can be represented by (7/3x, 7/3y)

What is Dilation in Transformation?

In mathematics, dilation can be defined as a transformation which alters the size of an object, though its shape remains unchanged. It is described as a specific similarity transformation where all dimensions associated with the given object (i.e. height, width and length) are increased or decreased uniformly by a particular scale factor.

A dilation thus produces an alteration in size, with the figure being either magnified or diminished while keeping its unique shape intact.

The scale factor used in the dilation is 9/7 instead of 7/3.

Learn more about transformation at

https://brainly.com/question/4289712

#SPJ1

Correct answer gets brainliest!!!!

Answers

Answer: I believe the only option would be Length.

The length determines the distance between the two endpoints.

I can't think of a reason why you'd need to use height or width when measuring a line on a graph. Besides if it were height, it'll just be the length of the line when vertical. And all generated graphed lines have the same width. You wouldn't need to measure it.

Hopefully it's correct.

Other Questions
Assume that prior to January 1, 2019, a Reporting Company owned a 15 percent interest in a Legal Entity. The Reporting Company acquired its 15 percent ownership interest in the Legal Entity on June 15, 1998 for $45,000, and correctly accounted for this investment under the cost method (i. E. , it was a passive investment and it was not marketable). On January 1, 2019, the Reporting Company purchased an additional 30 percent interest in the Legal Entity for $180,000. As a result of an evaluation of the facts and circumstances on Janu-ary 1, 2019, the Reporting Entity determined that the Legal Entity is a variable interest entity (VIE) and that the Reporting Company is the primary beneficiary of the VIE. The Reporting Company also determined that, on January 1, 2019, the fair value of the previously held 15 percent interest is $90,000. In addition, in-dependent appraisals revealed that the fair value of the noncontrolling interest (i. E. , the 55 percent not owned by the Reporting Company) is $330,000. On January 1, 2019, the Legal Entity has reported book values for its identifiable net assets equal to $399,000 and fair values for its identifiable net assets equal to $570,000. Assume that the Legal Entity is not a "business," as that term is defined in FASB ASC 805 ("Business Combinations"). Related to the initial consolidation of the Legal Entity on January 1, 2019, determine the following amounts:Note: Use a negative sign with your answer in part b. To indicate a gain on initial consolidation of Legal Entity, if applicable. AccountAmounta. GoodwillAnswerb. (Gain) Loss on initialconsolidation of Legal EntityAnswer What is the phrase that reflects the idea that the superpowers would avoid nuclear war due to fear of mutual annihilation?. The total weight of a shipping crate is modeled by the function c = 24b + 30, * where c is the total weight of the crate with b boxes packed inside the crate. If each crate holds a maximum of 6 boxes, then what are the domain and range of the function for this situation? 3. Let ya if (x,y) + (0,0) f(x,y) = x2 + y 0 if x=y=0. lim f(x,y) exist? Verify your claim. (x,y)+(0,0) (a) Does 4. Problems and Applications Q4 Consider two policies: a tax cut that will last for only one year and a tax cut that is expected to be permanent. True or False: A tax cut that is expected to be permanent will have a greater impact on aggregate demand than a tax cut that will last for only one year Many people eat portion sizes that are larger than the serving size listed on the nutritional information on the label Mary hears from a friend that green tea can provide extensive health benefits. On her way to work, she sees a billboard advertising the newest brand of green tea by Mildone. Later that day, at the supermarket, Mary sees a promotional offer on Mildone's green tea and decides to try out this brand to see whether green tea really has the benefits its users advocate. Which of the following statements is true with regard to this scenario?a. Mary's purchase of the green tea was independent of any marketinginfluence. b. Mary experienced group, marketing, and situational influences in thispurchase. c. Mary's experience of viewing the billboard was the group influence inthis scenario help nowwwwwwww PLSSSS Please describe this grapha. Explain the relationship between variables.b. State if it is a linear or nonlinear graph.c. Give an example of what this graph could be about. A pie graph uses _________to represent information. a. lines c. odd numbers b. dots d. percentages please select the best answer from the choices provided a b c d Differences in Will Eisners A Contract with God story and mainstream (such as Marvel DC, Fawcett, EC) comic books. whats the difference in both stories. Give your answer accurate to 3 decimal places.Claire starts at point A and runs east at a rate of 12 ft/sec. One minute later, Anna starts at A and runs north at a rate of 7 ft/sec. At what rate (in feet per second) is the distance between them changing after another minute?______ft/sec The measures of the interior angles of a hexagon are represented by , and. The measure of the largest interior angle is The type of economy most closely associated with the United States is a client is being seen in the clinic after receiving an external breast prosthesis after a mastectomy. what question from the nurse best evaluates the effectiveness of the prosthesis on body image A sequence can be generated using an + 1 = 0.25 + an, where a1 = 5 and n is a whole number greater than 1. What are the first 5 terms in the sequence? during the first month of operations ended may 31, big sky creations company produced 55,750 designer cowboy boots, of which 52,650 were sold. operating data for the month are summarized as follows: 1 sales $789,750.00 2 manufacturing costs: 3 direct materials $395,825.00 4 direct labor 117,075.00 5 variable manufacturing cost 61,325.00 6 fixed manufacturing cost 55,750.00 629,975.00 7 selling and administrative expenses: 8 variable $26,325.00 9 fixed 26,325.00 52,650.00 during june, big sky creations produced 49,550 designer cowboy boots and sold 52,650 cowboy boots. operating data for june are summarized as follows: 1 sales $789,750.00 2 manufacturing costs: 3 direct materials $351,805.00 4 direct labor 104,055.00 5 variable manufacturing cost 54,505.00 6 fixed manufacturing cost 55,750.00 566,115.00 7 selling and administrative expenses: 8 variable $26,325.00 9 fixed 26,325.00 52,650.00 required: 1. using the absorption costing concept, prepare income statements for (a) may and (b) june. 2. using the variable costing concept, prepare income statements for (a) may and (b) june. 3a. explain the reason for the differences in operating income in (1) and (2) for may. 3b. explain the reason for the differences in operating income in (1) and (2) for june. 4. based on your answers to (1) and (2), did big sky creations company operate more profitably in may or in june? explain. Dialogue between two students on the topic harmfull effects of drug addiction Rihanna is reviewing her classmates essay. Read this sentence from the essay:When students must wear mandatory uniforms, this policy negatively impacts their individuality, finances, and impacts behavior. Help Rihanna revise the sentence to create parallel structure. When students must wear mandatory uniforms, this policy negatively impacts their individuality, finances, and their behavior During this reaction, water is evaporating from the solution at the same time some of the co2 is dissolving into the water. How might these factors affect the results of the experiment? explain each effect and the overall effect.